Cossenos Racionais

Uma vez que \(\cos(\theta)=-\cos(\pi-\theta)\), para completar o argumento anterior falta apenas provar que:

\(\mathcal{P}_2:\) Quando \(\theta \in\,\,]0,\pi/2[\) é um múltiplo racional de \(\pi\) e tem cosseno racional, então \(\theta=\pi/3\).

A propriedade de existirem naturais \(p\), \(q\) tais que \(\theta= \frac{p\,\pi}{q}\) permite-nos calcular facilmente \(\cos(q\,\theta)\) pois vale \(\pm 1\). Mas como se relaciona \(\cos(q\,\theta)\) com \(\cos(\theta)\)? É o que começaremos por analisar.

Dados um ângulo \(x\) e um número natural \(n\), então \[ 2\,\cos\,(n+1)\,x =(2\,\cos\,n\,x)\,(2\,\cos\,x) - 2\,\cos\,(n-1)\,x\;\;\;(1)\] igualdade que é imediata a partir das fórmulas conhecidas para o cosseno de uma soma e de uma diferença de ângulos\[\begin{eqnarray*} \cos\,(z+w) &=& \cos\,z \,\cos w - sen \,z \,sen \, w \\ \cos\,(z-w) &=& \cos\,z \,\cos w + sen \,z \,sen \, w \end{eqnarray*}\] que, quando somadas, indicam que \[\cos\,(z+w) = 2\,\cos\,z \,\cos w - \cos\,(z-w).\] Basta agora considerar \(z=n\,x\), \(w=x\), obtendo-se\[\cos\,(n+1)\,x =2\,\cos\,n\,x\,\cos\,x - \cos\,(n-1)\,x\] e depois multiplicar esta igualdade por \(2\).

Desta fórmula, podemos deduzir facilmente outra propriedade de \(2\,\cos\,(n+1)\,x\): este valor pode ser expresso como uma combinação finita, com coeficientes inteiros, de potências de \(2\,\cos\,x\). Isto é, existem inteiros \(c_{n-1}, \,\cdots,\, c_1,\, c_0\) tais que\[ 2\,\cos\,n\,x = (2\,\cos\,x)^n + c_{n-1}\,(2\,\cos\,x)^{n-1} + \cdots + c_1 \,(2\,\cos\,x) + c_0.\;\;\;(2) \] Fixado um ângulo \(x\), demonstremos esta igualdade por indução em \(n \in \mathbb{N}\).

Para \(n=1\), a equação anterior é trivial, pede apenas que \(2\,\cos\,x = 2\,\cos\,x\).

Para \(n=2\), a equação é consequência directa de \(\cos\,2\,x=2\,\cos^2\,x - 1\), que se pode reescrever como \(2\,\cos\,2\,x = (2\,\cos\,x)^2 -2\).

Suponhamos agora que a igualdade \((2)\) é válida para \(n\) e \(n-1\), sendo \(n\) um natural fixado. Ou seja, admitamos que se tem, \[\begin{eqnarray*} 2\,\cos\,n\,x &=& (2\,\cos\,x)^{n} + a_{n-1}\,(2\,\cos\,x)^{n-1} + \cdots + a_1 \,(2\,\cos\,x) + a_0 \\ 2\,\cos\,(n-1)\,x &=& (2\,\cos\,x)^{n-1} + b_{n-2}\,(2\,\cos\,x)^{n-2} + \cdots + b_1 \,(2\,\cos\,x) + b_0 \end{eqnarray*}\] onde os coeficientes \(a_i's\) e \(b_j's\) são todos números inteiros. Queremos provar que uma expressão análoga vale para \(2\,\cos\,(n+1)\,x\). Para isso, voltemos à igualdade \((1)\) e reescrevamo-la do seguinte modo: \[\begin{eqnarray*} 2\,\cos\,(n+1)\,x &=& (2\,\cos\,n\,x)\,(2\,\cos\,x) - 2\,\cos\,(n-1)\,x = \\ &=& 2\,\cos\,x\,[(2\,\cos\,x)^{n} + a_{n-1}\,(2\,\cos\,x)^{n-1} + \cdots + a_0] -\\ &-& [(2\,\cos\,x)^{n-1} + b_{n-2}\,(2\,\cos\,x)^{n-2} + \cdots + b_1 \,(2\,\cos\,x) + b_0] \\ &=& (2\,\cos\,x)^{n+1} + a_{n-1}\,(2\,\cos\,x)^{n} + (a_{n-2}-1)\,(2\,\cos\,x)^{n-1} + \\ &+& (a_{n-3}-b_{n-2})\,(2\,\cos\,x)^{n-2} + \cdots + (a_0-b_1)(2\,\cos\,x) - b_0. \end{eqnarray*}\] Como todos os coeficientes \(c_k\) desta última soma são inteiros, o Princípio de Indução Finita garante agora que a propriedade \((2)\) é válida para \(x\) e todo o natural \(n\). Sendo \(x \in \mathbb{R}\) arbitrário, a prova da igualdade \((2)\) está completa.

Estamos em condições de terminar a prova da afirmação \(\mathcal{P}_2\).

Se \(\theta\) é um ângulo cuja amplitude é um múltiplo racional de \(\pi\), digamos \(\theta= \frac{p\,\pi}{q}\) para um par de naturais \(p,\) \(q\), então \(\cos(2q\,\theta)= 1\). Substituindo este valor na equação \((2)\), para \(n=2q\) e \(x=\theta\), obtemos \[2\,\cos\,2q\,\theta = (2\,\cos\,\theta)^{2q} + c_{2q-1}\,(2\,\cos\,\theta)^{2q-1} + \cdots + c_1 \,(2\,\cos\,\theta) + c_0\] ou seja,\[(2\,\cos\,\theta)^{2q} + c_{2q-1}\,(2\,\cos\,\theta)^{2q-1} + \cdots + c_1 \,(2\,\cos\,\theta) + c_0 - 2=0.\] Isto indica que \(2\,\cos\,\theta\), que por hipótese é um número racional, é um zero da função polinomial de coeficientes inteiros e grau \(2q\) cujo coeficiente de maior grau é \(1\), nomeadamente, \[t \in \mathbb{R} \,\, \mapsto \,\, t^{2q} + c_{2q-1}\,t^{2q-1} + \cdots + c_1 \,t + c_0 - 2.\] Ora bem, se \(\frac{r}{s}\) é uma fracção irredutível (isto é, \(r \in \mathbb{Z}\), \(s \in \mathbb{N}\) e o máximo divisor comum entre \(r\) e \(s\) é \(1\)) e \(\frac{r}{s}\) é um zero desta função polinomial, então \(\frac{r}{s}\) tem de ser um inteiro, uma vez que da igualdade \[\left(\frac{r}{s}\right)^{2q} + c_{2q-1}\,\left(\frac{r}{s}\right)^{2q-1} + \cdots + c_1 \,\left(\frac{r}{s}\right) + c_0 - 2=0\] concluímos que \[r^{2q} + c_{2q-1}\,r^{2q-1}\,s + \cdots + c_1 \,r\,s^{2q-1} + (c_0 - 2)s^{2q} = 0\] e, por \(c_{2q-1}, \,c_{2q-2},\,\cdots,\,c_{1},\,c_0-2\) serem inteiros, que o natural \(s\) tem de dividir \(r^{2q}\). Tendo em conta que \(r\) e \(s\) são primos entre si, devemos ter \(s=1\). Ou seja, \(2\,\cos\,\theta\) é um inteiro.

Sendo \(2\,\cos\,\theta\) um inteiro, restam-nos poucas possibilidades para o seu valor: como \(|\cos\,\theta|\leq 1\), tem-se \(-2\leq 2\,\cos\,\theta \leq 2\); além disso, como \(\theta \in\,\,]0,\pi/2[\), devemos ter \(0 < \cos\,\theta <1\), logo \(2\,\cos\,\theta \in \,\,]0, 2[\); mas neste intervalo só mora um inteiro, \(1\). Consequentemente, \(2\,\cos\,\theta=1\) e, portanto, \(\theta=\pi/3\).